LSAT and Law School Admissions Forum

Get expert LSAT preparation and law school admissions advice from PowerScore Test Preparation.

User avatar
 Dave Killoran
PowerScore Staff
  • PowerScore Staff
  • Posts: 5972
  • Joined: Mar 25, 2011
|
#87498
Complete Question Explanation
(The complete setup for this game can be found here: lsat/viewtopic.php?t=2097)

The correct answer choice is (C)

The best strategy on List questions is to use the individual rules of the game to eliminate incorrect answer choices.

Answer choice (A) can be eliminated because from the fourth rule if R does not volunteer then L must volunteer.

Answer choice (B) can be eliminated because from the second rule if M volunteers then T must volunteer. Alternatively, this answer choice can be eliminated because from the third rule if S does not volunteer then V must volunteer.

Answer choice (D) can be eliminated because from the first rule if R volunteers then M must volunteer.

Answer choice (E) can be eliminated because from the last rule if T volunteers then V cannot volunteer. Note that S and V can volunteer together, as this occurrence is not precluded by the third rule.

Answer choice (C) is thus proven correct by process of elimination.
 Basia W
  • Posts: 108
  • Joined: Jun 19, 2014
|
#16266
Good morning,

I just wanted to to go over my process to make sure I understood the question correctly: A) is incorrect because of the NOT S :arrow: V rule which stipulates that one or the other or both variables may appear, and E is incorrect because of the NOT R :arrow: L rule since one or the other or both can appear therefore not being "complete."

Thank you for your time,

best,

Basia
 Ron Gore
PowerScore Staff
  • PowerScore Staff
  • Posts: 220
  • Joined: May 15, 2013
|
#16267
Hi Basia,

Thanks for your question!

You ask:
Basia W wrote:A) is incorrect because of the NOT S :arrow: V rule which stipulates that one or the other or both variables may appear
First, let's tighten up your understanding of the S and V rule. That rule says that "if S does not volunteer, then V does." We diagram this as:

S :arrow: V

and the contrapositive is

V :arrow: S

This rule means that we MUST have AT LEAST ONE of S or V. We cannot have both of them NOT volunteer. The way this rule would be violated is if neither S nor V volunteers.

The rule about R and L operates the same way, and it would violate that rule to have neither R nor L volunteer.

Answer choice (A) is incorrect because it violates the R and L rule. The list in answer choice (A) contains neither R nor L, so it cannot be true.
Basia W wrote:E is incorrect because of the NOT R :arrow: L rule since one or the other or both can appear therefore not being "complete."


Answer choice (E) is incorrect, because it you can't have T and V volunteer together (last rule). The R and L rule is satisfied so long as one of the two (R or L) is present.

Please let me know if that helps.

Ron
 Basia W
  • Posts: 108
  • Joined: Jun 19, 2014
|
#16272
Hello,

yes that was very helpful!

thank you,

best,

Basia

Get the most out of your LSAT Prep Plus subscription.

Analyze and track your performance with our Testing and Analytics Package.